How Do I Find the Asymptotes of a Function?

  • Thread starter Thread starter donjt81
  • Start date Start date
Click For Summary
To find the asymptotes of the function y=(4x)/(x^2+1), first determine the horizontal asymptotes by comparing the degrees of the numerator and denominator. If the degree of the numerator (m) is less than that of the denominator (k), the horizontal asymptote is y=0; if they are equal, the asymptote is at a/b, where a and b are the leading coefficients. For vertical asymptotes, identify values of x that make the denominator zero, as these indicate where the function approaches infinity. The discussion also touches on the importance of understanding the degrees of the polynomial in the numerator and denominator. Clarification on terms like "complex asymptotes" and variable definitions is also sought.
donjt81
Messages
70
Reaction score
0
ok so i have this problem where i am asked to find the asymptotes. It is kinda throwing me off because it is in the middle of the differentiation section. so here is the problem

problem: use the graphing strategy to sketch the graph of y=(4x)/(x^2+1). check for domain values, intercepts, asymptotes, critical values, interval where the function is increasing and where it is decreasing, intervals where it is concave up and where it is concave down. Then graph it. please use sign charts.

I have done all the other stuff but I don't know how to find the asymptotes. Can someone help please.
 
Physics news on Phys.org
To find the horizonal asmptotes, you need to first factor the f(x) or y function (already done in your case), and then look at the co-efficients of the largest power of x in the numberator and denominator. f(x) = ax^m / bx^k. Then if
1.m < k, the asymptote is at y= 0.
2. m = k, the asymptote is at a/b
3. m > k, there is no asymptote

For vertical asymptotes, you need to find where the function goes to infinity (i.e. the value of x for which the denominator equals zero). I'm not sure what a complex asymptotes means.
 
Last edited:
what is k?
 
Just edited it, n = k.
 
Question: A clock's minute hand has length 4 and its hour hand has length 3. What is the distance between the tips at the moment when it is increasing most rapidly?(Putnam Exam Question) Answer: Making assumption that both the hands moves at constant angular velocities, the answer is ## \sqrt{7} .## But don't you think this assumption is somewhat doubtful and wrong?

Similar threads

Replies
11
Views
2K
  • · Replies 4 ·
Replies
4
Views
3K
  • · Replies 3 ·
Replies
3
Views
2K
  • · Replies 4 ·
Replies
4
Views
2K
  • · Replies 6 ·
Replies
6
Views
1K
  • · Replies 4 ·
Replies
4
Views
7K
  • · Replies 3 ·
Replies
3
Views
2K
  • · Replies 10 ·
Replies
10
Views
2K
  • · Replies 1 ·
Replies
1
Views
4K
  • · Replies 2 ·
Replies
2
Views
995